Difference between revisions of "1991 AHSME Problems"

m (Problem 27)
 
(21 intermediate revisions by 10 users not shown)
Line 1: Line 1:
 +
{{AHSME Problems|year = 1991}}
 +
 
== Problem 1 ==
 
== Problem 1 ==
  
 
If for any three distinct numbers <math>a</math>, <math>b</math>, and <math>c</math> we define <math>f(a,b,c)=\frac{c+a}{c-b}</math>, then <math>f(1,-2,-3)</math> is
 
If for any three distinct numbers <math>a</math>, <math>b</math>, and <math>c</math> we define <math>f(a,b,c)=\frac{c+a}{c-b}</math>, then <math>f(1,-2,-3)</math> is
  
<math> \textbf {(A) } -2 \qquad \textbf {(B) } -\frac{2}{5} \qquad \textbf {(C) } -\frac{1}{4} \qquad \textbf {(D) } \frac{2}{5} \qquad \textbf {(E) } 2 </math>
+
<math> \textbf{(A) } -2 \qquad \textbf{(B) } -\frac{2}{5} \qquad \textbf{(C) } -\frac{1}{4} \qquad \textbf{(D) } \frac{2}{5} \qquad \textbf {(E) } 2 </math>
 
 
  
 
[[1991 AHSME Problems/Problem 1|Solution]]
 
[[1991 AHSME Problems/Problem 1|Solution]]
Line 12: Line 13:
 
<math>|3-\pi|=</math>
 
<math>|3-\pi|=</math>
  
<math> \textbf{(A)\ }\frac{1}{7}\qquad\textbf{(B)\ }0.14\qquad\textbf{(C)\ }3-\pi\qquad\textbf{(D)\ }3+\pi\qquad\textbf{(E)\ }\pi-3 </math>
+
<math> \textbf{(A) }\frac{1}{7} \qquad \textbf{(B) }0.14 \qquad \textbf{(C) }3-\pi \qquad \textbf{(D) }3+\pi \qquad \textbf{(E) }\pi-3 </math>
 
 
  
 
[[1991 AHSME Problems/Problem 2|Solution]]
 
[[1991 AHSME Problems/Problem 2|Solution]]
Line 21: Line 21:
 
<math>(4^{-1}-3^{-1})^{-1}=</math>
 
<math>(4^{-1}-3^{-1})^{-1}=</math>
  
(A) <math>-12</math>  (B) <math>-1</math>  (C) <math>\frac{1}{12}</math>  (D) <math>1</math>  (E) <math>12</math>
+
<math> \textbf{(A) }-12 \qquad \textbf{(B) }-1 \qquad \textbf{(C) }\frac{1}{12} \qquad \textbf{(D) }1 \qquad \textbf{(E) }12 </math>
  
 
[[1991 AHSME Problems/Problem 3|Solution]]
 
[[1991 AHSME Problems/Problem 3|Solution]]
Line 29: Line 29:
 
Which of the following triangles cannot exist?
 
Which of the following triangles cannot exist?
  
(A) An acute isosceles triangle (B) An isosceles right triangle (C) An obtuse right triangle (D) A scalene right triangle (E) A scalene obtuse tr
+
<math>\textbf{(A) }</math> An acute isosceles triangle
iangle
+
 
 +
<math>\textbf{(B) }</math> An isosceles right triangle
  
 +
<math>\textbf{(C) }</math> An obtuse right triangle
 +
 +
<math>\textbf{(D) }</math> A scalene right triangle
 +
 +
<math>\textbf{(E) }</math> A scalene obtuse triangle
  
 
[[1991 AHSME Problems/Problem 4|Solution]]
 
[[1991 AHSME Problems/Problem 4|Solution]]
  
 
== Problem 5 ==
 
== Problem 5 ==
 
 
<asy>
 
<asy>
 
draw((0,0)--(2,2)--(2,1)--(5,1)--(5,-1)--(2,-1)--(2,-2)--cycle,dot);
 
draw((0,0)--(2,2)--(2,1)--(5,1)--(5,-1)--(2,-1)--(2,-2)--cycle,dot);
Line 44: Line 49:
  
 
In the arrow-shaped polygon [see figure], the angles at vertices <math>A,C,D,E</math> and <math>F</math> are right angles, <math>BC=FG=5, CD=FE=20, DE=10</math>, and <math>AB=AG</math>. The area of the polygon is closest to
 
In the arrow-shaped polygon [see figure], the angles at vertices <math>A,C,D,E</math> and <math>F</math> are right angles, <math>BC=FG=5, CD=FE=20, DE=10</math>, and <math>AB=AG</math>. The area of the polygon is closest to
<math>\text{(A) } 288\quad
+
 
\text{(B) } 291\quad
+
<math>\textbf{(A) } 288\qquad\textbf{(B) } 291\qquad\textbf{(C) } 294\qquad\textbf{(D) } 297\qquad\textbf{(E) } 300</math>
\text{(C) } 294\quad
 
\text{(D) } 297\quad
 
\text{(E) } 300</math>
 
  
 
[[1991 AHSME Problems/Problem 5|Solution]]
 
[[1991 AHSME Problems/Problem 5|Solution]]
  
 
== Problem 6 ==
 
== Problem 6 ==
 
  
 
If <math>x\geq 0</math>, then <math>\sqrt{x\sqrt{x\sqrt{x}}}=</math>
 
If <math>x\geq 0</math>, then <math>\sqrt{x\sqrt{x\sqrt{x}}}=</math>
  
(A) <math>x\sqrt{x}</math> (B) <math>x \sqrt[4]{x}</math> (C) <math>\sqrt[8]{x}</math> (D) <math>\sqrt[8]{x^3}</math> (E) <math>\sqrt[8]{x^7}</math>
+
<math>\textbf{(A) } x\sqrt{x}\qquad
 +
\textbf{(B) } x\sqrt[4]{x}\qquad
 +
\textbf{(C) } \sqrt[8]{x}\qquad
 +
\textbf{(D) } \sqrt[8]{x^3}\qquad
 +
\textbf{(E) } \sqrt[8]{x^7}</math>
  
 
[[1991 AHSME Problems/Problem 6|Solution]]
 
[[1991 AHSME Problems/Problem 6|Solution]]
Line 65: Line 70:
 
If <math>x=\frac{a}{b}</math>, <math>a\neq b</math> and <math>b\neq 0</math>, then <math>\frac{a+b}{a-b}=</math>
 
If <math>x=\frac{a}{b}</math>, <math>a\neq b</math> and <math>b\neq 0</math>, then <math>\frac{a+b}{a-b}=</math>
  
(A) <math>\frac{x}{x+1}</math> (B) <math>\frac{x+1}{x-1}</math> (C) <math>1</math> (D) <math>x-\frac{1}{x}</math> (E) <math>x+\frac{1}{x}</math>
+
<math>\textbf{(A) } \frac{x}{x+1} \qquad \textbf{(B) } \frac{x+1}{x-1} \qquad \textbf{(C) } 1 \qquad \textbf{(D) } x-\frac{1}{x} \qquad \textbf{(E) } x+\frac{1}{x}</math>
  
 
[[1991 AHSME Problems/Problem 7|Solution]]
 
[[1991 AHSME Problems/Problem 7|Solution]]
Line 73: Line 78:
 
Liquid <math>X</math> does not mix with water. Unless obstructed, it spreads out on the surface of water to form a circular film <math>0.1</math>cm thick. A rectangular box measuring <math>6</math>cm by <math>3</math>cm by <math>12</math>cm is filled with liquid <math>X</math>. Its contents are poured onto a large body of water. What will be the radius, in centimeters, of the resulting circular film?
 
Liquid <math>X</math> does not mix with water. Unless obstructed, it spreads out on the surface of water to form a circular film <math>0.1</math>cm thick. A rectangular box measuring <math>6</math>cm by <math>3</math>cm by <math>12</math>cm is filled with liquid <math>X</math>. Its contents are poured onto a large body of water. What will be the radius, in centimeters, of the resulting circular film?
  
(A) <math>\frac{\sqrt{216}}{\pi}</math> (B) <math>\sqrt{\frac{216}{\pi}}</math> (C) <math>\sqrt{\frac{2160}{\pi}}</math> (D) <math>\frac{216}{\pi}</math> (E) <math>\frac{2160}{\pi}</math>
+
<math>\textbf{(A) } \frac{\sqrt{216}}{\pi} \qquad \textbf{(B) }\sqrt{\frac{216}{\pi}} \qquad \textbf{(C) } \sqrt{\frac{2160}{\pi}} \qquad \textbf{(D) } \frac{216}{\pi} \qquad \textbf{(E) } \frac{2160}{\pi}</math>
 
 
  
 
[[1991 AHSME Problems/Problem 8|Solution]]
 
[[1991 AHSME Problems/Problem 8|Solution]]
Line 82: Line 86:
 
From time <math>t=0</math> to time <math>t=1</math> a population increased by <math>i\%</math>, and from time <math>t=1</math> to time <math>t=2</math> the population increased by <math>j\%</math>. Therefore, from time <math>t=0</math> to time <math>t=2</math> the population increased by
 
From time <math>t=0</math> to time <math>t=1</math> a population increased by <math>i\%</math>, and from time <math>t=1</math> to time <math>t=2</math> the population increased by <math>j\%</math>. Therefore, from time <math>t=0</math> to time <math>t=2</math> the population increased by
  
<math>\text{(A) (i+j)\%} \quad
+
<math>\textbf{(A) } (i+j)\% \qquad
\text{(B) } ij\%\quad
+
\textbf{(B) } ij\% \qquad
\text{(C) } (i+ij)\%\quad
+
\textbf{(C) } (i+ij)\% \qquad
\text{(D) } \left(i+j+\frac{ij}{100}\right)\%\quad
+
\textbf{(D) } \left(i+j+\frac{ij}{100}\right)\% \qquad
\text{(E) } \left(i+j+\frac{i+j}{100}\right)\%</math>
+
\textbf{(E) } \left(i+j+\frac{i+j}{100}\right)\%</math>
  
 
[[1991 AHSME Problems/Problem 9|Solution]]
 
[[1991 AHSME Problems/Problem 9|Solution]]
Line 94: Line 98:
 
Point <math>P</math> is <math>9</math> units from the center of a circle of radius <math>15</math>. How many different chords of the circle contain <math>P</math> and have integer lengths?
 
Point <math>P</math> is <math>9</math> units from the center of a circle of radius <math>15</math>. How many different chords of the circle contain <math>P</math> and have integer lengths?
  
(A) 11 (B) 12 (C) 13 (D) 14 (E) 29
+
<math>\textbf{(A) } 11\qquad
 +
\textbf{(B) } 12\qquad
 +
\textbf{(C) } 13\qquad
 +
\textbf{(D) } 14\qquad
 +
\textbf{(E) } 29</math>
  
 
[[1991 AHSME Problems/Problem 10|Solution]]
 
[[1991 AHSME Problems/Problem 10|Solution]]
  
 
== Problem 11 ==
 
== Problem 11 ==
Jack and Jill run 10 km. They start at the same point, run 5 km up a hill, and reurn to the starting point by the same route. Jack has a 10 minute head start and runs at the rate of 15 km/hr uphill and 20 km/hr downhill. Jill runs 16 km/hr uphill and 22 km/hr downhill. How far from the top of the hill are they when they pass each other going in opposite directions (in km)?
+
Jack and Jill run 10 km. They start at the same point, run 5 km up a hill, and return to the starting point by the same route. Jack has a 10 minute head start and runs at the rate of 15 km/hr uphill and 20 km/hr downhill. Jill runs 16 km/hr uphill and 22 km/hr downhill. How far from the top of the hill are they when they pass each other going in opposite directions (in km)?
  
<math>\text{(A) } \frac{5}{4}\quad
+
<math>\textbf{(A) } \frac{5}{4}\qquad
\text{(B) } \frac{35}{27}\quad
+
\textbf{(B) } \frac{35}{27}\qquad
\text{(C) } \frac{27}{20}\quad
+
\textbf{(C) } \frac{27}{20}\qquad
\text{(D) } \frac{7}{3}\quad
+
\textbf{(D) } \frac{7}{3}\qquad
\text{(E) } \frac{28}{49}</math>
+
\textbf{(E) } \frac{28}{49}</math>
  
 
[[1991 AHSME Problems/Problem 11|Solution]]
 
[[1991 AHSME Problems/Problem 11|Solution]]
Line 113: Line 121:
 
The measures (in degrees) of the interior angles of a convex hexagon form an arithmetic sequence of integers. Let <math>m</math> be the measure of the largest interior angle of the hexagon. The largest possible value of <math>m</math>, in degrees, is
 
The measures (in degrees) of the interior angles of a convex hexagon form an arithmetic sequence of integers. Let <math>m</math> be the measure of the largest interior angle of the hexagon. The largest possible value of <math>m</math>, in degrees, is
  
(A) 165 (B) 167 (C) 170 (D) 175 (E) 179
+
<math>\textbf{(A) } 165\qquad
 +
\textbf{(B) } 167\qquad
 +
\textbf{(C) } 170\qquad
 +
\textbf{(D) } 175\qquad
 +
\textbf{(E) } 179</math>
  
 
[[1991 AHSME Problems/Problem 12|Solution]]
 
[[1991 AHSME Problems/Problem 12|Solution]]
Line 121: Line 133:
 
Horses <math>X,Y</math> and <math>Z</math> are entered in a three-horse race in which ties are not possible. The odds against <math>X</math> winning are <math>3:1</math> and the odds against <math>Y</math> winning are <math>2:3</math>, what are the odds against <math>Z</math> winning? (By "odds against <math>H</math> winning are <math>p:q</math>" we mean the probability of <math>H</math> winning the race is <math>\frac{q}{p+q}</math>.)
 
Horses <math>X,Y</math> and <math>Z</math> are entered in a three-horse race in which ties are not possible. The odds against <math>X</math> winning are <math>3:1</math> and the odds against <math>Y</math> winning are <math>2:3</math>, what are the odds against <math>Z</math> winning? (By "odds against <math>H</math> winning are <math>p:q</math>" we mean the probability of <math>H</math> winning the race is <math>\frac{q}{p+q}</math>.)
  
<math>\text{(A) } 3:20\quad
+
<math>\textbf{(A) } 3:20\qquad
\text{(B) } 5:6\quad
+
\textbf{(B) } 5:6\qquad
\text{(C) } 8:5\quad
+
\textbf{(C) } 8:5\qquad
\text{(D) } 17:3\quad
+
\textbf{(D) } 17:3\qquad
\text{(E) } 20:3</math>
+
\textbf{(E) } 20:3</math>
  
 
[[1991 AHSME Problems/Problem 13|Solution]]
 
[[1991 AHSME Problems/Problem 13|Solution]]
  
 
== Problem 14 ==
 
== Problem 14 ==
 
  
 
If <math>x</math> is the cube of a positive integer and <math>d</math> is the number of positive integers that are divisors of <math>x</math>, then <math>d</math> could be
 
If <math>x</math> is the cube of a positive integer and <math>d</math> is the number of positive integers that are divisors of <math>x</math>, then <math>d</math> could be
  
(A) <math>200</math>  (B) <math>201</math>  (C) <math>202</math>  (D) <math>203</math>  (E) <math>204</math>
+
<math>\textbf{(A) } 200\qquad
 +
\textbf{(B) } 201\qquad
 +
\textbf{(C) } 202\qquad
 +
\textbf{(D) } 203\qquad
 +
\textbf{(E) } 204</math>
  
 
[[1991 AHSME Problems/Problem 14|Solution]]
 
[[1991 AHSME Problems/Problem 14|Solution]]
Line 142: Line 157:
 
A circular table has 60 chairs around it. There are <math>N</math> people seated at this table in such a way that the next person seated must sit next to someone. What is the smallest possible value for <math>N</math>?
 
A circular table has 60 chairs around it. There are <math>N</math> people seated at this table in such a way that the next person seated must sit next to someone. What is the smallest possible value for <math>N</math>?
  
<math>\text{(A) } 15\quad
+
<math>\textbf{(A) } 15\qquad
\text{(B) } 20\quad
+
\textbf{(B) } 20\qquad
\text{(C) } 30\quad
+
\textbf{(C) } 30\qquad
\text{(D) } 40\quad
+
\textbf{(D) } 40\qquad
\text{(E) } 58</math>
+
\textbf{(E) } 58</math>
  
 
[[1991 AHSME Problems/Problem 15|Solution]]
 
[[1991 AHSME Problems/Problem 15|Solution]]
Line 154: Line 169:
 
One hundred students at Century High School participated in the AHSME last year, and their mean score was 100. The number of non-seniors taking the AHSME was <math>50\%</math> more than the number of seniors, and the mean score of the seniors was <math>50\%</math> higher than that of the non-seniors. What was the mean score of the seniors?
 
One hundred students at Century High School participated in the AHSME last year, and their mean score was 100. The number of non-seniors taking the AHSME was <math>50\%</math> more than the number of seniors, and the mean score of the seniors was <math>50\%</math> higher than that of the non-seniors. What was the mean score of the seniors?
  
(A) <math>100</math> (B) <math>112.5</math> (C) <math>120</math> (D) <math>125</math> (E) <math>150</math>
+
<math>\textbf{(A) } 100\qquad
 +
\textbf{(B) } 112.5\qquad
 +
\textbf{(C) } 120\qquad
 +
\textbf{(D) } 125\qquad
 +
\textbf{(E) } 150</math>
  
 
[[1991 AHSME Problems/Problem 16|Solution]]
 
[[1991 AHSME Problems/Problem 16|Solution]]
Line 167: Line 186:
 
How many years in the millenium between 1000 and 2000 have properties (a) and (b)?
 
How many years in the millenium between 1000 and 2000 have properties (a) and (b)?
  
<math>\text{(A) } 1\quad
+
<math>\textbf{(A) } 1\qquad
\text{(B) } 2\quad
+
\textbf{(B) } 2\qquad
\text{(C) } 3\quad
+
\textbf{(C) } 3\qquad
\text{(D) } 4\quad
+
\textbf{(D) } 4\qquad
\text{(E) } 5</math>
+
\textbf{(E) } 5</math>
  
 
[[1991 AHSME Problems/Problem 17|Solution]]
 
[[1991 AHSME Problems/Problem 17|Solution]]
Line 179: Line 198:
 
If <math>S</math> is the set of points <math>z</math> in the complex plane such that <math>(3+4i)z</math> is a real number, then <math>S</math> is a  
 
If <math>S</math> is the set of points <math>z</math> in the complex plane such that <math>(3+4i)z</math> is a real number, then <math>S</math> is a  
  
(A) right triangle  
+
<math>\textbf{(A) }</math> right triangle <math>\qquad \textbf{(B) }</math> circle <math>\qquad \textbf{(C) }</math> hyperbola <math>\qquad \textbf{(D) }</math> line <math>\qquad \textbf{(E) }</math> parabola
(B) circle
 
(C) hyperbola
 
(D) line
 
(E) parabola
 
 
 
  
 
[[1991 AHSME Problems/Problem 18|Solution]]
 
[[1991 AHSME Problems/Problem 18|Solution]]
Line 199: Line 213:
 
Triangle <math>ABC</math> has a right angle at <math>C, AC=3</math> and <math>BC=4</math>. Triangle <math>ABD</math> has a right angle at <math>A</math> and <math>AD=12</math>. Points <math>C</math> and <math>D</math> are on opposite sides of <math>\overline{AB}</math>. The line through <math>D</math> parallel to <math>\overline{AC}</math> meets <math>\overline{CB}</math> extended at <math>E</math>. If
 
Triangle <math>ABC</math> has a right angle at <math>C, AC=3</math> and <math>BC=4</math>. Triangle <math>ABD</math> has a right angle at <math>A</math> and <math>AD=12</math>. Points <math>C</math> and <math>D</math> are on opposite sides of <math>\overline{AB}</math>. The line through <math>D</math> parallel to <math>\overline{AC}</math> meets <math>\overline{CB}</math> extended at <math>E</math>. If
 
<cmath>\frac{DE}{DB}=\frac{m}{n},</cmath>
 
<cmath>\frac{DE}{DB}=\frac{m}{n},</cmath>
where <math>m</math> and <math>n</math> are relatively prime positive integers, then <math>m+n=</math>
+
where <math>m</math> and <math>n</math> are relatively prime positive integers, then <math>m+n</math> is
  
<math>\text{(A) } 25\quad
+
<math>\textbf{(A) } 25\qquad
\text{(B) } 128\quad
+
\textbf{(B) } 128\qquad
\text{(C) } 153\quad
+
\textbf{(C) } 153\qquad
\text{(D) } 243\quad
+
\textbf{(D) } 243\qquad
\text{(E) } 256</math>
+
\textbf{(E) } 256</math>
  
 
[[1991 AHSME Problems/Problem 19|Solution]]
 
[[1991 AHSME Problems/Problem 19|Solution]]
Line 213: Line 227:
 
The sum of all real <math>x</math> such that <math>(2^x-4)^3+(4^x-2)^3=(4^x+2^x-6)^3</math> is
 
The sum of all real <math>x</math> such that <math>(2^x-4)^3+(4^x-2)^3=(4^x+2^x-6)^3</math> is
  
(A) 3/2  (B) 2 (C) 5/2  (D) 3 (E) 7/2
+
<math>\textbf{(A) } \frac32 \qquad \textbf{(B) } 2 \qquad \textbf{(C) } \frac52 \qquad \textbf{(D) } 3 \qquad \textbf{(E) } \frac72</math>
  
 
[[1991 AHSME Problems/Problem 20|Solution]]
 
[[1991 AHSME Problems/Problem 20|Solution]]
Line 220: Line 234:
  
  
For all real numbers <math>x</math> except <math>x=0</math> and <math>x=1</math> the function <math>f(x)</math> is defined by <math>f(x/(1-x))=1/x</math>. Suppose <math>0\leq t\leq \pi/2</math>. What is the value of <math>f(\sec^2t)</math>?
+
For all real numbers <math>x</math> except <math>x=0</math> and <math>x=1</math> the function <math>f(x)</math> is defined by <math>f(x/(x-1))=1/x</math>. Suppose <math>0\leq t\leq \pi/2</math>. What is the value of <math>f(\sec^2t)</math>?
  
<math>\text{(A) } sin^2\theta\quad
+
<math>\textbf{(A) } \sin^2\theta\qquad
\text{(B) } cos^2\theta\quad
+
\textbf{(B) } \cos^2\theta\qquad
\text{(C) } tan^2\theta\quad
+
\textbf{(C) } \tan^2\theta\qquad
\text{(D) } cot^2\theta\quad
+
\textbf{(D) } \cot^2\theta\qquad
\text{(E) } csc^2\theta</math>
+
\textbf{(E) } \csc^2\theta</math>
  
 
[[1991 AHSME Problems/Problem 21|Solution]]
 
[[1991 AHSME Problems/Problem 21|Solution]]
Line 240: Line 254:
 
MP("P",(0,0),S);
 
MP("P",(0,0),S);
 
</asy>
 
</asy>
 
  
 
Two circles are externally tangent. Lines <math>\overline{PAB}</math> and <math>\overline{PA'B'}</math> are common tangents with <math>A</math> and <math>A'</math> on the smaller circle <math>B</math> and <math>B'</math> on the larger circle. If <math>PA=AB=4</math>, then the area of the smaller circle is
 
Two circles are externally tangent. Lines <math>\overline{PAB}</math> and <math>\overline{PA'B'}</math> are common tangents with <math>A</math> and <math>A'</math> on the smaller circle <math>B</math> and <math>B'</math> on the larger circle. If <math>PA=AB=4</math>, then the area of the smaller circle is
  
<math>\text{(A) } 1.44\pi\quad
+
<math>\textbf{(A) } 1.44\pi\qquad
\text{(B) } 2\pi\quad
+
\textbf{(B) } 2\pi\qquad
\text{(C) } 2.56\pi\quad
+
\textbf{(C) } 2.56\pi\qquad
\text{(D) } \sqrt{8}\pi\quad
+
\textbf{(D) } \sqrt{8}\pi\qquad
\text{(E) } 4\pi</math>
+
\textbf{(E) } 4\pi</math>
 
 
  
 
[[1991 AHSME Problems/Problem 22|Solution]]
 
[[1991 AHSME Problems/Problem 22|Solution]]
Line 264: Line 276:
 
</asy>
 
</asy>
  
 +
If <math>ABCD</math> is a <math>2\times2</math> square, <math>E</math> is the midpoint of <math>\overline{AB}</math>,<math>F</math> is the midpoint of <math>\overline{BC}</math>,<math>\overline{AF}</math> and <math>\overline{DE}</math> intersect at <math>I</math>, and <math>\overline{BD}</math> and <math>\overline{AF}</math> intersect at <math>H</math>, then the area of quadrilateral <math>BEIH</math> is
  
If <math>ABCD</math> is a <math>2X2</math> square, <math>E</math> is the midpoint of <math>\overline{AB}</math>,<math>F</math> is the midpoint of <math>\overline{BC}</math>,<math>\overline{AF}</math> and <math>\overline{DE}</math> intersect at <math>I</math>, and <math>\overline{BD}</math> and <math>\overline{AF}</math> intersect at <math>H</math>, then the area of quadrilateral <math>BEIH</math> is
+
<math>\textbf{(A) } \frac{1}{3}\qquad
 
+
\textbf{(B) } \frac{2}{5}\qquad
<math>\text{(A) } \frac{1}{3}\quad
+
\textbf{(C) } \frac{7}{15}\qquad
\text{(B) } \frac{2}{5}\quad
+
\textbf{(D) } \frac{8}{15}\qquad
\text{(C) } \frac{7}{15}\quad
+
\textbf{(E) } \frac{3}{5}</math>
\text{(D) } \frac{8}{15}\quad
 
\text{(E) } \frac{3}{5}</math>
 
  
 
[[1991 AHSME Problems/Problem 23|Solution]]
 
[[1991 AHSME Problems/Problem 23|Solution]]
Line 279: Line 290:
 
The graph, <math>G</math> of <math>y=\log_{10}x</math> is rotated <math>90^{\circ}</math> counter-clockwise about the origin to obtain a new graph <math>G'</math>. Which of the following is an equation for <math>G'</math>?
 
The graph, <math>G</math> of <math>y=\log_{10}x</math> is rotated <math>90^{\circ}</math> counter-clockwise about the origin to obtain a new graph <math>G'</math>. Which of the following is an equation for <math>G'</math>?
  
(A) <math>y=\log_{10}\left(\frac{x+90}{9}\right)</math> (B) <math>y=\log_{x}10</math> (C) <math>y=\frac{1}{x+1}</math> (D) <math>y=10^{-x}</math> (E) <math>y=10^x</math>
+
<math>\textbf{(A) } y=\log_{10}\left(\frac{x+90}{9}\right) \qquad
 +
\textbf{(B) } y=\log_{x}10 \qquad
 +
\textbf{(C) } y=\frac{1}{x+1} \qquad
 +
\textbf{(D) } y=10^{-x} \qquad
 +
\textbf{(E) } y=10^x</math>
  
 
[[1991 AHSME Problems/Problem 24|Solution]]
 
[[1991 AHSME Problems/Problem 24|Solution]]
Line 289: Line 304:
 
for <math>n=2,3,4,\cdots,</math> then <math>P_{1991}</math> is closest to which of the following numbers?
 
for <math>n=2,3,4,\cdots,</math> then <math>P_{1991}</math> is closest to which of the following numbers?
  
<math>\text{(A) } 2.0\quad
+
<math>\textbf{(A) } 2.0\qquad
\text{(B) } 2.3\quad
+
\textbf{(B) } 2.3\qquad
\text{(C) } 2.6\quad
+
\textbf{(C) } 2.6\qquad
\text{(D) } 2.9\quad
+
\textbf{(D) } 2.9\qquad
\text{(E) } 3.2</math>
+
\textbf{(E) } 3.2</math>
  
 
[[1991 AHSME Problems/Problem 25|Solution]]
 
[[1991 AHSME Problems/Problem 25|Solution]]
Line 300: Line 315:
  
 
An <math>n</math>-digit positive integer is cute if its <math>n</math> digits are an arrangement of the set <math>\{1,2,...,n\}</math> and its first  
 
An <math>n</math>-digit positive integer is cute if its <math>n</math> digits are an arrangement of the set <math>\{1,2,...,n\}</math> and its first  
<math>k</math> digits form an integer that is divisible by <math>k</math>  , for  <math>k  = 1,2,...,n</math>. For example, <math>321</math> is a cute <math>3</math>-digit integer because <math>1</math> divides <math>3</math>, <math>2</math> divides <math>32</math>, and <math>3</math> divides <math>321</math>. Howmany cute <math>6</math>-digit integers are there?
+
<math>k</math> digits form an integer that is divisible by <math>k</math>  , for  <math>k  = 1,2,...,n</math>. For example, <math>321</math> is a cute <math>3</math>-digit integer because <math>1</math> divides <math>3</math>, <math>2</math> divides <math>32</math>, and <math>3</math> divides <math>321</math>. How many cute <math>6</math>-digit integers are there?
 
 
<math>\text{(A) } 0\quad
 
\text{(B) } 1\quad
 
\text{(C) } 2\quad
 
\text{(D) } 3\quad
 
\text{(E) } 4</math>
 
  
 +
<math>\textbf{(A) } 0\qquad
 +
\textbf{(B) } 1\qquad
 +
\textbf{(C) } 2\qquad
 +
\textbf{(D) } 3\qquad
 +
\textbf{(E) } 4</math>
  
 
[[1991 AHSME Problems/Problem 26|Solution]]
 
[[1991 AHSME Problems/Problem 26|Solution]]
Line 313: Line 327:
 
== Problem 27 ==
 
== Problem 27 ==
  
If <math>x+\sqrt{x^2-1}+\frac{1}{x-\sqrt{x^2-1}}=20</math> then <math>x^2+\sqrt{x^4-1}+\frac{1}{x^2+\sqrt{x^4-1}}=</math>
+
If <cmath>x+\sqrt{x^2-1}+\frac{1}{x-\sqrt{x^2-1}}=20,</cmath> then <cmath>x^2+\sqrt{x^4-1}+\frac{1}{x^2+\sqrt{x^4-1}}=</cmath>
 
 
(A) <math>5.05</math> (B) <math>20</math> (C) <math>51.005</math> (D) <math>61.25</math> (E) <math>400</math>
 
  
 +
<math>\textbf{(A) } 5.05 \qquad
 +
\textbf{(B) } 20 \qquad
 +
\textbf{(C) } 51.005 \qquad
 +
\textbf{(D) } 61.25 \qquad
 +
\textbf{(E) } 400</math>
  
 
[[1991 AHSME Problems/Problem 27|Solution]]
 
[[1991 AHSME Problems/Problem 27|Solution]]
Line 324: Line 341:
 
Initially an urn contains 100 white and 100 black marbles. Repeatedly 3 marbles are removed (at random) from the urn and replaced with some marbles from a pile outside the urn as follows: 3 blacks are replaced with 1 black, or 2 blacks and 1 white are replaced with a white and a black, or 1 black and 2 whites are replaced with 2 whites, or 3 whites are replaced with a black and a white. Which of the following could be the contents of the urn after repeated applications of this procedure?
 
Initially an urn contains 100 white and 100 black marbles. Repeatedly 3 marbles are removed (at random) from the urn and replaced with some marbles from a pile outside the urn as follows: 3 blacks are replaced with 1 black, or 2 blacks and 1 white are replaced with a white and a black, or 1 black and 2 whites are replaced with 2 whites, or 3 whites are replaced with a black and a white. Which of the following could be the contents of the urn after repeated applications of this procedure?
  
(A) 2 black (B) 2 white (C) 1 black (D) 1 black and 1 white (E) 1 white
+
<math>\textbf{(A) }</math> 2 black <math>\qquad \textbf{(B) }</math> 2 white <math>\qquad \textbf{(C) }</math> 1 black <math>\qquad \textbf{(D) }</math> 1 black and 1 white <math>\qquad \textbf{(E) }</math> 1 white
  
 
[[1991 AHSME Problems/Problem 28|Solution]]
 
[[1991 AHSME Problems/Problem 28|Solution]]
Line 332: Line 349:
 
Equilateral triangle <math>ABC</math> has <math>P</math> on <math>AB</math> and <math>Q</math> on <math>AC</math>. The triangle is folded along <math>PQ</math> so that vertex <math>A</math> now rests at <math>A'</math> on side <math>BC</math>. If <math>BA'=1</math> and <math>A'C=2</math> then the length of the crease <math>PQ</math> is
 
Equilateral triangle <math>ABC</math> has <math>P</math> on <math>AB</math> and <math>Q</math> on <math>AC</math>. The triangle is folded along <math>PQ</math> so that vertex <math>A</math> now rests at <math>A'</math> on side <math>BC</math>. If <math>BA'=1</math> and <math>A'C=2</math> then the length of the crease <math>PQ</math> is
  
(A) <math>\frac{8}{5}</math> (B) <math>\frac{7}{20}\sqrt{21}</math> (C) <math>\frac{1+\sqrt{5}}{2}</math> (D) <math>\frac{13}{8}</math> (E) <math>\sqrt{3}</math>
+
<math>\textbf{(A) } \frac{8}{5} \qquad
 +
\textbf{(B) } \frac{7}{20}\sqrt{21} \qquad
 +
\textbf{(C) } \frac{1+\sqrt{5}}{2} \qquad
 +
\textbf{(D) } \frac{13}{8} \qquad
 +
\textbf{(E) } \sqrt{3}</math>
  
 
[[1991 AHSME Problems/Problem 29|Solution]]
 
[[1991 AHSME Problems/Problem 29|Solution]]
  
 
== Problem 30 ==
 
== Problem 30 ==
 
  
 
For any set <math>S</math>, let <math>|S|</math> denote the number of elements in <math>S</math>, and let <math>n(S)</math> be the number of subsets of <math>S</math>, including the empty set and the set <math>S</math> itself. If <math>A</math>, <math>B</math>, and <math>C</math> are sets for which <math>n(A)+n(B)+n(C)=n(A\cup B\cup C)</math> and <math>|A|=|B|=100</math>, then what is the minimum possible value of <math>|A\cap B\cap C|</math>?
 
For any set <math>S</math>, let <math>|S|</math> denote the number of elements in <math>S</math>, and let <math>n(S)</math> be the number of subsets of <math>S</math>, including the empty set and the set <math>S</math> itself. If <math>A</math>, <math>B</math>, and <math>C</math> are sets for which <math>n(A)+n(B)+n(C)=n(A\cup B\cup C)</math> and <math>|A|=|B|=100</math>, then what is the minimum possible value of <math>|A\cap B\cap C|</math>?
  
(A) 96 (B) 97 (C) 98 (D) 99 (E) 100
+
<math>\textbf{(A) } 96 \qquad \textbf{(B) } 97 \qquad \textbf{(C) } 98 \qquad \textbf{(D) } 99 \qquad \textbf{(E) } 100</math>
  
 
[[1991 AHSME Problems/Problem 30|Solution]]
 
[[1991 AHSME Problems/Problem 30|Solution]]
  
 +
== See also ==
 +
 +
* [[AMC 12 Problems and Solutions]]
 +
* [[Mathematics competition resources]]
 +
 +
{{AHSME box|year=1991|before=[[1990 AHSME]]|after=[[1992 AHSME]]}} 
  
== See also ==
 
*[[AHSME]]
 
*[[AMC12]]
 
 
{{MAA Notice}}
 
{{MAA Notice}}

Latest revision as of 04:27, 6 September 2021

1991 AHSME (Answer Key)
Printable versions: WikiAoPS ResourcesPDF

Instructions

  1. This is a 30-question, multiple choice test. Each question is followed by answers marked A, B, C, D and E. Only one of these is correct.
  2. You will receive 5 points for each correct answer, 2 points for each problem left unanswered, and 0 points for each incorrect answer.
  3. No aids are permitted other than scratch paper, graph paper, ruler, compass, protractor and erasers.
  4. Figures are not necessarily drawn to scale.
  5. You will have 90 minutes working time to complete the test.
1 2 3 4 5 6 7 8 9 10 11 12 13 14 15 16 17 18 19 20 21 22 23 24 25 26 27 28 29 30

Problem 1

If for any three distinct numbers $a$, $b$, and $c$ we define $f(a,b,c)=\frac{c+a}{c-b}$, then $f(1,-2,-3)$ is

$\textbf{(A) } -2 \qquad \textbf{(B) } -\frac{2}{5} \qquad \textbf{(C) } -\frac{1}{4} \qquad \textbf{(D) } \frac{2}{5} \qquad \textbf {(E) } 2$

Solution

Problem 2

$|3-\pi|=$

$\textbf{(A) }\frac{1}{7} \qquad \textbf{(B) }0.14 \qquad \textbf{(C) }3-\pi \qquad \textbf{(D) }3+\pi \qquad \textbf{(E) }\pi-3$

Solution

Problem 3

$(4^{-1}-3^{-1})^{-1}=$

$\textbf{(A) }-12 \qquad \textbf{(B) }-1 \qquad \textbf{(C) }\frac{1}{12} \qquad \textbf{(D) }1 \qquad \textbf{(E) }12$

Solution

Problem 4

Which of the following triangles cannot exist?

$\textbf{(A) }$ An acute isosceles triangle

$\textbf{(B) }$ An isosceles right triangle

$\textbf{(C) }$ An obtuse right triangle

$\textbf{(D) }$ A scalene right triangle

$\textbf{(E) }$ A scalene obtuse triangle

Solution

Problem 5

[asy] draw((0,0)--(2,2)--(2,1)--(5,1)--(5,-1)--(2,-1)--(2,-2)--cycle,dot); MP("A",(0,0),W);MP("B",(2,2),N);MP("C",(2,1),S);MP("D",(5,1),NE);MP("E",(5,-1),SE);MP("F",(2,-1),NW);MP("G",(2,-2),S); MP("5",(2,1.5),E);MP("5",(2,-1.5),E);MP("20",(3.5,1),N);MP("20",(3.5,-1),S);MP("10",(5,0),E); [/asy]

In the arrow-shaped polygon [see figure], the angles at vertices $A,C,D,E$ and $F$ are right angles, $BC=FG=5, CD=FE=20, DE=10$, and $AB=AG$. The area of the polygon is closest to

$\textbf{(A) } 288\qquad\textbf{(B) } 291\qquad\textbf{(C) } 294\qquad\textbf{(D) } 297\qquad\textbf{(E) } 300$

Solution

Problem 6

If $x\geq 0$, then $\sqrt{x\sqrt{x\sqrt{x}}}=$

$\textbf{(A) } x\sqrt{x}\qquad \textbf{(B) } x\sqrt[4]{x}\qquad \textbf{(C) } \sqrt[8]{x}\qquad \textbf{(D) } \sqrt[8]{x^3}\qquad \textbf{(E) } \sqrt[8]{x^7}$

Solution

Problem 7

If $x=\frac{a}{b}$, $a\neq b$ and $b\neq 0$, then $\frac{a+b}{a-b}=$

$\textbf{(A) } \frac{x}{x+1} \qquad \textbf{(B) } \frac{x+1}{x-1} \qquad \textbf{(C) } 1 \qquad \textbf{(D) } x-\frac{1}{x} \qquad \textbf{(E) } x+\frac{1}{x}$

Solution

Problem 8

Liquid $X$ does not mix with water. Unless obstructed, it spreads out on the surface of water to form a circular film $0.1$cm thick. A rectangular box measuring $6$cm by $3$cm by $12$cm is filled with liquid $X$. Its contents are poured onto a large body of water. What will be the radius, in centimeters, of the resulting circular film?

$\textbf{(A) } \frac{\sqrt{216}}{\pi} \qquad \textbf{(B) }\sqrt{\frac{216}{\pi}} \qquad \textbf{(C) } \sqrt{\frac{2160}{\pi}} \qquad \textbf{(D) } \frac{216}{\pi} \qquad \textbf{(E) } \frac{2160}{\pi}$

Solution

Problem 9

From time $t=0$ to time $t=1$ a population increased by $i\%$, and from time $t=1$ to time $t=2$ the population increased by $j\%$. Therefore, from time $t=0$ to time $t=2$ the population increased by

$\textbf{(A) } (i+j)\% \qquad \textbf{(B) } ij\% \qquad \textbf{(C) } (i+ij)\% \qquad \textbf{(D) } \left(i+j+\frac{ij}{100}\right)\% \qquad \textbf{(E) } \left(i+j+\frac{i+j}{100}\right)\%$

Solution

Problem 10

Point $P$ is $9$ units from the center of a circle of radius $15$. How many different chords of the circle contain $P$ and have integer lengths?

$\textbf{(A) } 11\qquad \textbf{(B) } 12\qquad \textbf{(C) } 13\qquad \textbf{(D) } 14\qquad \textbf{(E) } 29$

Solution

Problem 11

Jack and Jill run 10 km. They start at the same point, run 5 km up a hill, and return to the starting point by the same route. Jack has a 10 minute head start and runs at the rate of 15 km/hr uphill and 20 km/hr downhill. Jill runs 16 km/hr uphill and 22 km/hr downhill. How far from the top of the hill are they when they pass each other going in opposite directions (in km)?

$\textbf{(A) } \frac{5}{4}\qquad \textbf{(B) } \frac{35}{27}\qquad \textbf{(C) } \frac{27}{20}\qquad \textbf{(D) } \frac{7}{3}\qquad \textbf{(E) } \frac{28}{49}$

Solution

Problem 12

The measures (in degrees) of the interior angles of a convex hexagon form an arithmetic sequence of integers. Let $m$ be the measure of the largest interior angle of the hexagon. The largest possible value of $m$, in degrees, is

$\textbf{(A) } 165\qquad \textbf{(B) } 167\qquad \textbf{(C) } 170\qquad \textbf{(D) } 175\qquad \textbf{(E) } 179$

Solution

Problem 13

Horses $X,Y$ and $Z$ are entered in a three-horse race in which ties are not possible. The odds against $X$ winning are $3:1$ and the odds against $Y$ winning are $2:3$, what are the odds against $Z$ winning? (By "odds against $H$ winning are $p:q$" we mean the probability of $H$ winning the race is $\frac{q}{p+q}$.)

$\textbf{(A) } 3:20\qquad \textbf{(B) } 5:6\qquad \textbf{(C) } 8:5\qquad \textbf{(D) } 17:3\qquad \textbf{(E) } 20:3$

Solution

Problem 14

If $x$ is the cube of a positive integer and $d$ is the number of positive integers that are divisors of $x$, then $d$ could be

$\textbf{(A) } 200\qquad \textbf{(B) } 201\qquad \textbf{(C) } 202\qquad \textbf{(D) } 203\qquad \textbf{(E) } 204$

Solution

Problem 15

A circular table has 60 chairs around it. There are $N$ people seated at this table in such a way that the next person seated must sit next to someone. What is the smallest possible value for $N$?

$\textbf{(A) } 15\qquad \textbf{(B) } 20\qquad \textbf{(C) } 30\qquad \textbf{(D) } 40\qquad \textbf{(E) } 58$

Solution

Problem 16

One hundred students at Century High School participated in the AHSME last year, and their mean score was 100. The number of non-seniors taking the AHSME was $50\%$ more than the number of seniors, and the mean score of the seniors was $50\%$ higher than that of the non-seniors. What was the mean score of the seniors?

$\textbf{(A) } 100\qquad \textbf{(B) } 112.5\qquad \textbf{(C) } 120\qquad \textbf{(D) } 125\qquad \textbf{(E) } 150$

Solution

Problem 17

A positive integer $N$ is a palindrome if the integer obtained by reversing the sequence of digits of $N$ is equal to $N$. The year 1991 is the only year in the current century with the following 2 properties:

(a) It is a palindrome (b) It factors as a product of a 2-digit prime palindrome and a 3-digit prime palindrome.

How many years in the millenium between 1000 and 2000 have properties (a) and (b)?

$\textbf{(A) } 1\qquad \textbf{(B) } 2\qquad \textbf{(C) } 3\qquad \textbf{(D) } 4\qquad \textbf{(E) } 5$

Solution

Problem 18

If $S$ is the set of points $z$ in the complex plane such that $(3+4i)z$ is a real number, then $S$ is a

$\textbf{(A) }$ right triangle $\qquad \textbf{(B) }$ circle $\qquad \textbf{(C) }$ hyperbola $\qquad \textbf{(D) }$ line $\qquad \textbf{(E) }$ parabola

Solution

Problem 19

[asy] draw((0,0)--(0,3)--(4,0)--cycle,dot); draw((4,0)--(7,0)--(7,10)--cycle,dot); draw((0,3)--(7,10),dot); MP("C",(0,0),SW);MP("A",(0,3),NW);MP("B",(4,0),S);MP("E",(7,0),SE);MP("D",(7,10),NE); [/asy]

Triangle $ABC$ has a right angle at $C, AC=3$ and $BC=4$. Triangle $ABD$ has a right angle at $A$ and $AD=12$. Points $C$ and $D$ are on opposite sides of $\overline{AB}$. The line through $D$ parallel to $\overline{AC}$ meets $\overline{CB}$ extended at $E$. If \[\frac{DE}{DB}=\frac{m}{n},\] where $m$ and $n$ are relatively prime positive integers, then $m+n$ is

$\textbf{(A) } 25\qquad \textbf{(B) } 128\qquad \textbf{(C) } 153\qquad \textbf{(D) } 243\qquad \textbf{(E) } 256$

Solution

Problem 20

The sum of all real $x$ such that $(2^x-4)^3+(4^x-2)^3=(4^x+2^x-6)^3$ is

$\textbf{(A) } \frac32 \qquad \textbf{(B) } 2 \qquad \textbf{(C) } \frac52 \qquad \textbf{(D) } 3 \qquad \textbf{(E) } \frac72$

Solution

Problem 21

For all real numbers $x$ except $x=0$ and $x=1$ the function $f(x)$ is defined by $f(x/(x-1))=1/x$. Suppose $0\leq t\leq \pi/2$. What is the value of $f(\sec^2t)$?

$\textbf{(A) } \sin^2\theta\qquad \textbf{(B) } \cos^2\theta\qquad \textbf{(C) } \tan^2\theta\qquad \textbf{(D) } \cot^2\theta\qquad \textbf{(E) } \csc^2\theta$

Solution

Problem 22

[asy] draw(circle((0,6sqrt(2)),2sqrt(2)),black+linewidth(.75)); draw(circle((0,3sqrt(2)),sqrt(2)),black+linewidth(.75)); draw((-8/3,16sqrt(2)/3)--(-4/3,8sqrt(2)/3)--(0,0)--(4/3,8sqrt(2)/3)--(8/3,16sqrt(2)/3),dot); MP("B",(-8/3,16*sqrt(2)/3),W);MP("B'",(8/3,16*sqrt(2)/3),E); MP("A",(-4/3,8*sqrt(2)/3),W);MP("A'",(4/3,8*sqrt(2)/3),E); MP("P",(0,0),S); [/asy]

Two circles are externally tangent. Lines $\overline{PAB}$ and $\overline{PA'B'}$ are common tangents with $A$ and $A'$ on the smaller circle $B$ and $B'$ on the larger circle. If $PA=AB=4$, then the area of the smaller circle is

$\textbf{(A) } 1.44\pi\qquad \textbf{(B) } 2\pi\qquad \textbf{(C) } 2.56\pi\qquad \textbf{(D) } \sqrt{8}\pi\qquad \textbf{(E) } 4\pi$

Solution

Problem 23

[asy] draw((0,0)--(0,2)--(2,2)--(2,0)--cycle,dot); draw((2,2)--(0,0)--(0,1)--cycle,dot); draw((0,2)--(1,0),dot); MP("B",(0,0),SW);MP("A",(0,2),NW);MP("D",(2,2),NE);MP("C",(2,0),SE); MP("E",(0,1),W);MP("F",(1,0),S);MP("H",(2/3,2/3),E);MP("I",(2/5,6/5),N); dot((1,0));dot((0,1));dot((2/3,2/3));dot((2/5,6/5)); [/asy]

If $ABCD$ is a $2\times2$ square, $E$ is the midpoint of $\overline{AB}$,$F$ is the midpoint of $\overline{BC}$,$\overline{AF}$ and $\overline{DE}$ intersect at $I$, and $\overline{BD}$ and $\overline{AF}$ intersect at $H$, then the area of quadrilateral $BEIH$ is

$\textbf{(A) } \frac{1}{3}\qquad \textbf{(B) } \frac{2}{5}\qquad \textbf{(C) } \frac{7}{15}\qquad \textbf{(D) } \frac{8}{15}\qquad \textbf{(E) } \frac{3}{5}$

Solution

Problem 24

The graph, $G$ of $y=\log_{10}x$ is rotated $90^{\circ}$ counter-clockwise about the origin to obtain a new graph $G'$. Which of the following is an equation for $G'$?

$\textbf{(A) } y=\log_{10}\left(\frac{x+90}{9}\right) \qquad \textbf{(B) } y=\log_{x}10 \qquad \textbf{(C) } y=\frac{1}{x+1} \qquad \textbf{(D) } y=10^{-x} \qquad \textbf{(E) } y=10^x$

Solution

Problem 25

If $T_n=1+2+3+\cdots +n$ and \[P_n=\frac{T_2}{T_2-1}\cdot\frac{T_3}{T_3-1}\cdot\frac{T_4}{T_4-1}\cdot\cdots\cdot\frac{T_n}{T_n-1}\] for $n=2,3,4,\cdots,$ then $P_{1991}$ is closest to which of the following numbers?

$\textbf{(A) } 2.0\qquad \textbf{(B) } 2.3\qquad \textbf{(C) } 2.6\qquad \textbf{(D) } 2.9\qquad \textbf{(E) } 3.2$

Solution

Problem 26

An $n$-digit positive integer is cute if its $n$ digits are an arrangement of the set $\{1,2,...,n\}$ and its first $k$ digits form an integer that is divisible by $k$ , for $k  = 1,2,...,n$. For example, $321$ is a cute $3$-digit integer because $1$ divides $3$, $2$ divides $32$, and $3$ divides $321$. How many cute $6$-digit integers are there?

$\textbf{(A) } 0\qquad \textbf{(B) } 1\qquad \textbf{(C) } 2\qquad \textbf{(D) } 3\qquad \textbf{(E) } 4$

Solution

Problem 27

If \[x+\sqrt{x^2-1}+\frac{1}{x-\sqrt{x^2-1}}=20,\] then \[x^2+\sqrt{x^4-1}+\frac{1}{x^2+\sqrt{x^4-1}}=\]

$\textbf{(A) } 5.05 \qquad \textbf{(B) } 20 \qquad \textbf{(C) } 51.005 \qquad \textbf{(D) } 61.25 \qquad \textbf{(E) } 400$

Solution

Problem 28

Initially an urn contains 100 white and 100 black marbles. Repeatedly 3 marbles are removed (at random) from the urn and replaced with some marbles from a pile outside the urn as follows: 3 blacks are replaced with 1 black, or 2 blacks and 1 white are replaced with a white and a black, or 1 black and 2 whites are replaced with 2 whites, or 3 whites are replaced with a black and a white. Which of the following could be the contents of the urn after repeated applications of this procedure?

$\textbf{(A) }$ 2 black $\qquad \textbf{(B) }$ 2 white $\qquad \textbf{(C) }$ 1 black $\qquad \textbf{(D) }$ 1 black and 1 white $\qquad \textbf{(E) }$ 1 white

Solution

Problem 29

Equilateral triangle $ABC$ has $P$ on $AB$ and $Q$ on $AC$. The triangle is folded along $PQ$ so that vertex $A$ now rests at $A'$ on side $BC$. If $BA'=1$ and $A'C=2$ then the length of the crease $PQ$ is

$\textbf{(A) } \frac{8}{5} \qquad \textbf{(B) } \frac{7}{20}\sqrt{21} \qquad \textbf{(C) } \frac{1+\sqrt{5}}{2} \qquad \textbf{(D) } \frac{13}{8} \qquad \textbf{(E) } \sqrt{3}$

Solution

Problem 30

For any set $S$, let $|S|$ denote the number of elements in $S$, and let $n(S)$ be the number of subsets of $S$, including the empty set and the set $S$ itself. If $A$, $B$, and $C$ are sets for which $n(A)+n(B)+n(C)=n(A\cup B\cup C)$ and $|A|=|B|=100$, then what is the minimum possible value of $|A\cap B\cap C|$?

$\textbf{(A) } 96 \qquad \textbf{(B) } 97 \qquad \textbf{(C) } 98 \qquad \textbf{(D) } 99 \qquad \textbf{(E) } 100$

Solution

See also

1991 AHSME (ProblemsAnswer KeyResources)
Preceded by
1990 AHSME
Followed by
1992 AHSME
1 2 3 4 5 6 7 8 9 10 11 12 13 14 15 16 17 18 19 20 21 22 23 24 25 26 27 28 29 30
All AHSME Problems and Solutions


The problems on this page are copyrighted by the Mathematical Association of America's American Mathematics Competitions. AMC logo.png